The risk-free rate is 6.4 percent and the market risk premium is 5 percent. Assume that required returns are based on the CAPM.

The risk-free rate is 6.4 percent and the market risk premium is 5 percent. Assume that required returns are based on the CAPM. Your $1 million portfolio consists of $ 336 ,000 invested in a stock that has a beta of 1.7 and the remainder invested in a stock that has a beta of 0.8 . What is the required return on this portfolio? Enter your answer to the nearest .1%. Do not use the % sign in your answer, thus 12.1% is 12. 1 rather than 12.1% or .121.

0 replies

Leave a Reply

Want to join the discussion?
Feel free to contribute!

Leave a Reply

Your email address will not be published. Required fields are marked *